Difference between revisions of "2008 AIME I Problems/Problem 14"
m (→Problem) |
(→Solution 1) |
||
(3 intermediate revisions by 3 users not shown) | |||
Line 29: | Line 29: | ||
Let <math>k = \frac{2x-27}{x^2} \Longrightarrow kx^2 - 2x + 27 = 0</math>; this is a quadratic, and its [[discriminant]] must be nonnegative: <math>(-2)^2 - 4(k)(27) \ge 0 \Longleftrightarrow k \le \frac{1}{27}</math>. Thus, | Let <math>k = \frac{2x-27}{x^2} \Longrightarrow kx^2 - 2x + 27 = 0</math>; this is a quadratic, and its [[discriminant]] must be nonnegative: <math>(-2)^2 - 4(k)(27) \ge 0 \Longleftrightarrow k \le \frac{1}{27}</math>. Thus, | ||
<cmath>BP^2 \le 405 + 729 \cdot \frac{1}{27} = \boxed{432}</cmath> | <cmath>BP^2 \le 405 + 729 \cdot \frac{1}{27} = \boxed{432}</cmath> | ||
− | Equality holds when <math>x = 27</math>. | + | Equality holds when <math>x = 27</math>.~Shen Kislay Kai |
==== Solution 1.1 (Calculus) ==== | ==== Solution 1.1 (Calculus) ==== | ||
Line 42: | Line 42: | ||
Note: Please edit this solution if it feels inadequate. | Note: Please edit this solution if it feels inadequate. | ||
+ | ~Shen Kislay Kai | ||
===Solution 2=== | ===Solution 2=== | ||
Line 107: | Line 108: | ||
(Diagram credit goes to Solution 2) | (Diagram credit goes to Solution 2) | ||
− | We let <math>AC=x</math>. From similar triangles, we have that <math>PC=\frac{x\sqrt{x^2+18x}}{x+9}</math>. Similarly, <math>TP=QT=\frac{9\sqrt{x^2+18x}}{x+9}</math>. Using the Pythagorean Theorem, <math>BQ=\sqrt{(x+18)^2-(\frac{(x+18)\sqrt{x^2+18x}}{x+9})^2}</math>. Using the Pythagorean Theorem | + | We let <math>AC=x</math>. From similar triangles, we have that <math>PC=\frac{x\sqrt{x^2+18x}}{x+9}</math> (Use Pythagorean on <math>\triangle\omega TC</math> and then using <math>\triangle\omega CT\sim\triangle ACP</math>). Similarly, <math>TP=QT=\frac{9\sqrt{x^2+18x}}{x+9}</math>. Using the Pythagorean Theorem again and <math>\triangle CAP\sim\triangle CBQ</math>, <math>BQ=\sqrt{(x+18)^2-(\frac{(x+18)\sqrt{x^2+18x}}{x+9})^2}</math>. Using the Pythagorean Theorem <math>\bold{again}</math>, <math>BP=\sqrt{(x+18)^2-(\frac{(x+18)\sqrt{x^2+18x}}{x+9})^2+(\frac{18\sqrt{x^2+18x}}{x+9})^2}</math>. After a large bashful simplification, <math>BP=\sqrt{405+\frac{1458x-6561}{x^2+18x+81}}</math>. The fraction is equivalent to <math>729\frac{2x-9}{(x+9)^2}</math>. Taking the derivative of the fraction and solving for x, we get that <math>x=18</math>. Plugging <math>x=18</math> back into the expression for <math>BP</math> yields <math>\sqrt{432}</math>, so the answer is <math>(\sqrt{432})^2=\boxed{432}</math>. |
===Solution 4=== | ===Solution 4=== | ||
Line 143: | Line 144: | ||
~superagh | ~superagh | ||
+ | |||
+ | ===Solution 5 (Clean)=== | ||
+ | Let <math>h</math> be the distance from <math>A</math> to <math>CT</math>. Observe that <math>h</math> takes any value from <math>0</math> to <math>2r</math>, where <math>r</math> is the radius of the circle. | ||
+ | |||
+ | Let <math>Q</math> be the foot of the altitude from <math>B</math> to <math>CT</math>. It is clear that <math>T</math> is the midpoint of <math>PQ</math>, and so the length <math>OT</math> is the average of <math>AP</math> and <math>BQ</math>. It follows thus that <math>BQ = 2r - h</math>. | ||
+ | |||
+ | We compute <math>PT = \sqrt{r^2 - (r - h)^2} = \sqrt{h(2r - h)},</math> | ||
+ | and so <math>BP^2 = PQ^2 + BQ^2 = 4PT^2 + BQ^2 = 4h(2r - h) + (2r-h)^2 = (2r-h)(2r + 3h)</math>. | ||
+ | This is <math>\frac{1}{3}(6r - 3h)(2r + 3h) \le \frac{1}{3} \cdot \left( \frac{8r}{2} \right)^2</math>. Equality is attained, so thus we extract the answer of <math>\frac{16 \cdot 9^2}{3} = 27 \cdot 16 = \boxed{432}.</math> | ||
== See also == | == See also == |
Latest revision as of 12:29, 3 September 2024
Contents
Problem
Let be a diameter of circle . Extend through to . Point lies on so that line is tangent to . Point is the foot of the perpendicular from to line . Suppose , and let denote the maximum possible length of segment . Find .
Solution
Solution 1
Let . Since , it follows easily that . Thus . By the Law of Cosines on , where , so: Let ; this is a quadratic, and its discriminant must be nonnegative: . Thus, Equality holds when .~Shen Kislay Kai
Solution 1.1 (Calculus)
Proceed as follows for Solution 1.
Once you approach the function , find the maximum value by setting .
Simplifying to take the derivative, we have , so . Setting , we have .
Solving, we obtain as the critical value. Hence, has the maximum value of . Since , the maximum value of occurs at , so has a maximum value of .
Note: Please edit this solution if it feels inadequate. ~Shen Kislay Kai
Solution 2
From the diagram, we see that , and that .
This is a quadratic equation, maximized when . Thus, .
Solution 3 (Calculus Bash)
(Diagram credit goes to Solution 2)
We let . From similar triangles, we have that (Use Pythagorean on and then using ). Similarly, . Using the Pythagorean Theorem again and , . Using the Pythagorean Theorem , . After a large bashful simplification, . The fraction is equivalent to . Taking the derivative of the fraction and solving for x, we get that . Plugging back into the expression for yields , so the answer is .
Solution 4
(Diagram credit goes to Solution 2)
Let . The only constraint on is that it must be greater than . Using similar triangles, we can deduce that . Now, apply law of cosines on . We can see that . We can find . Plugging this into our equation, we get: Eventually, We want to maximize . There are many ways to maximize this expression, discussed here: https://artofproblemsolving.com/community/c4h2292700_maximization. The maximum result of that expression is . Finally, evaluating for this value .
~superagh
Solution 5 (Clean)
Let be the distance from to . Observe that takes any value from to , where is the radius of the circle.
Let be the foot of the altitude from to . It is clear that is the midpoint of , and so the length is the average of and . It follows thus that .
We compute and so . This is . Equality is attained, so thus we extract the answer of
See also
2008 AIME I (Problems • Answer Key • Resources) | ||
Preceded by Problem 13 |
Followed by Problem 15 | |
1 • 2 • 3 • 4 • 5 • 6 • 7 • 8 • 9 • 10 • 11 • 12 • 13 • 14 • 15 | ||
All AIME Problems and Solutions |
The problems on this page are copyrighted by the Mathematical Association of America's American Mathematics Competitions.